CIC Certification Pre-test Flashcards

1
Q

Which of the following statements about the susceptibility of antimicrobial-resistant bacteria to disinfectants is true?

a. Antimicrobial-resistant bacteria are more susceptible to disinfectants than non-resistant bacteria.

b. Antimicrobial-resistant bacteria are less susceptible to disinfectants than non-resistant bacteria.

c. Antimicrobial-resistant bacteria equally susceptible to disinfectants as non-resistant bacteria.

d. Antimicrobial-resistant bacteria require higher levels of disinfectant to become inactivated than antimicrobial-sensitive bacteria.

A

Rationale
c. While microbes may develop reduced susceptibility to disinfectants, this is not a concern from a practical standpoint because the concentrations of disinfectants used in practice are still sufficient to be effective against even less susceptible microbes. Antimicrobial-resistant bacteria are no more likely to develop reduced susceptibility to disinfectants than antimicrobial-sensitive bacteria are.

How well did you know this?
1
Not at all
2
3
4
5
Perfectly
2
Q

What is the purpose of bacterial biofilm formation?

a. It prevents adherence to the cell wall.

b. Biofilm kills competing organisms.

c. It is the cell’s preferred method of growth.

d. Biofilm decreases the organism’s concentration.

A

Rationale
c. Biofilm supports bacterial growth, increasing their concentration by attaching to cell walls.

How well did you know this?
1
Not at all
2
3
4
5
Perfectly
3
Q

A patient presents to your facility with a month-long cough, weight loss, fever, night sweats, and hemoptysis. A sputum Gram stain is reported as “many white blood cells (WBCs), no organisms seen”; and an acid-fast bacilli (AFB) sputum smear is reported as “many AFB seen.” The chest x-ray shows upper-lobe cavitary lesions. Which of the following organisms is the most likely cause of the patient’s symptoms?

a. Mycobacterium fortuitum

b. Mycobacterium abscessus

c. Mycobacterium chelonae

d. Mycobacterium tuberculosis

A

Rationale
d. Mycobacterium tuberculosis infections include symptoms of a cough lasting longer than three weeks, weight loss, fever, night sweats, and hemoptysis along with cavitary lesions on chest x-rays. M. abscessus and M. chelonae are associated with skin and soft tissue infections. M. fortuitum may cause infections and skin abscesses at the site of trauma and has been associated with nail salons.

How well did you know this?
1
Not at all
2
3
4
5
Perfectly
4
Q

Which of the following is best suited to minimizing exposure to infectious agents while handling soiled healthcare textiles?

a. Paper bags can be used to contain wet or soiled textiles.

b. Assume soiled healthcare textiles are contaminated and follow standard precautions (SP).

c. Sort soiled textiles and rinse them in patient-care areas.

d. Put clean linen from isolation rooms with other clean linen.

A

Rationale
b. To minimize exposure to infectious agents while handling soiled healthcare textiles, healthcare personnel (HCP) shouldn’t sort or rinse soiled textiles in patient-care areas. All all linen from isolation rooms should be treated as though it is soiled linen. Tear-resistant and leak-proof collection bags or containers should be used to contain wet or soiled textiles.

How well did you know this?
1
Not at all
2
3
4
5
Perfectly
5
Q

Which of the following educational methods is best suited to conveying education and training to adult learners?

a. Passive learning environment

b. Written goals or objectives for education

c. Directives without rationale

d. Active learning environment

A

Rationale
d. A number of different learning styles exist, but an active learning environment is best suited to adult learners because it allows the IP to engage them in a way that the information and training taught is more likely to be retained.

How well did you know this?
1
Not at all
2
3
4
5
Perfectly
6
Q

A research study explored and described the experiences and perceptions of infection preventionists (IPs) working in acute care hospitals during the coronavirus disease 2019 (COVID-19) global pandemic. What type of research study does this describe?

a. Case-control

b. Qualitative

c. Quantitative

d. Quasi-experimental

A

Rationale
b. Qualitative research gathers participants’ experiences, perceptions, and behavior. In contrast, quantitative research measures variables using a numerical system, analyzes them using any of a variety of statistical models, and reports relationships and associations among the studied variables. Quasi-experiments are studies that aim to evaluate interventions but that do not use randomization. A case-control study is a retrospective, observational study that compares two existing groups.

How well did you know this?
1
Not at all
2
3
4
5
Perfectly
7
Q

A dialysis patient is admitted to the hospital with a red, tender catheter insertion site. Which of the following is the most likely explanation for what happened to the patient?

a. They showered without covering the catheter site with an impermeable dressing.

b. They went out for dinner with some family members.

c. They visited a friend who is suffering from congestive heart failure.

d. They replaced the compromised catheter dressing as instructed.

A

Rationale
a. Patients should be instructed not to submerge their catheter or insertion site in water, but they may still shower if the catheter and insertion site are covered with an impermeable dressing. Replacing a compromised dressing should not cause an infection to develop nor should visiting friends or going out for dinner.

How well did you know this?
1
Not at all
2
3
4
5
Perfectly
8
Q

Which of the following precautions should be used for an immunocompromised patient suspected of having cryptococcal meningitis?

a. Airborne precautions for 24 hours after antibiotic is started if the patient is improving

b. Mask worn when within three feet from the patient’s bed

c. Standard precautions (SP) for staff and patient’s family

d. Contact precautions for staff; patient’s family restricted from visiting other patients

A

Rationale
c. Cryptococcosis, or cryptococcal disease, is a potentially fatal fungal disease caused by two species of Cryptococcus: C. neoformans and C. gattii. Cryptococcal meningitis is believed to result from dissemination of the fungus from either an observed or unappreciated pulmonary infection. Fungal meningitis is not transmitted from person to person and requires the use of SP.

How well did you know this?
1
Not at all
2
3
4
5
Perfectly
9
Q

An infection preventionist (IP) has identified an increase in catheter-associated urinary tract infections (CAUTIs) on the orthopedic unit. What are the action plan elements for reducing CAUTIs on this unit?

a. Device use, goals, strategy, person responsible, time frame, and measurements

b. Infection outcome/risk item, goals, strategy, person responsible, time frame, and measurements

c. Infection outcome/risk item, purpose, strategy, person responsible, time frame, and measurements

d. Infection outcome/risk item, goal, strategy, unit responsible, time frame, and measurements

A

Rationale

b. By developing a comprehensive surveillance plan, the IP can clearly identify actions needed to improve patient outcomes. The plan should include details about infection outcomes, plan goals and strategy, the person responsible, time frame, and measurements.

How well did you know this?
1
Not at all
2
3
4
5
Perfectly
10
Q

While performing routine surveillance, an infection preventionist (IP) identifies an alarming increase in the number of streptococcal infections, which are all related to one department in their facility. The rate of infection was above the expected norm for the population. Which of the following terms describes this situation?

a. Pandemic

b. Whole-house surveillance

c. Passive surveillance

d. Outbreak

A

Rationale

d. An outbreak is an increase in the number of identified cases above what is normal or expected for a given population.

How well did you know this?
1
Not at all
2
3
4
5
Perfectly
11
Q

An infection preventionist (IP) is evaluating a demolition project in an area of the hospital adjacent to the oncology department. Which of the following actions should the IP take to mitigate the risk?

a. Perform environmental sampling.

b. Consult with the engineering department to evaluate the air-handling system.

c. Recommend adding high-efficiency particulate air (HEPA) filers to the heating, ventilation, and air conditioning (HVAC) system.

d. Transfer all immunocompromised patients to another unit.

A

Rationale

b. Construction, renovation, repair, excavation, and demolition activities in hospitals and other healthcare facilities require planning and coordination among departments to minimize the infection risk for immunocompromised patients. The infection control risk assessment (ICRA) must be developed with the input of HVAC and plumbing engineers, architects, facility managers, medical staff, clinical department heads, safety specialists, and other individuals with a vested interest in the project.

How well did you know this?
1
Not at all
2
3
4
5
Perfectly
12
Q

Surgeons at a local hospital are hesitant to accept the benefits of patient normothermia during operative procedures. The surgery department chair asks the infection preventionist (IP) to perform a literature review of the evidence for perioperative normothermia. Which of the following levels of evidence should the IP present to the chair?

a. Case-control studies

b. Quasi-experimental studies

c. Randomized clinical trials

d. Case reports

A

Rationale

c. Among the options listed, randomized clinical trials are the most reliable because they provide the strongest evidence; they are, therefore, the most convincing. Only systemic reviews of multiple studies are considered more reliable.

How well did you know this?
1
Not at all
2
3
4
5
Perfectly
13
Q

Which of the following terms describes the harmfulness of a pathogen?

a. Pathogenicity

b. Inoculum

c. Virulence

d. Infectivity

A

Rationale

c. There are several characteristics that influence the transmissibility of an organism and its ability to cause disease. One of these is virulence, which is the ability of an organism to grow and multiply.

How well did you know this?
1
Not at all
2
3
4
5
Perfectly
14
Q

An infection preventionist (IP) is called to the day hospital care center for a possible outbreak of hepatitis A (HAV). The public health nurse is assisting the IP in investigating the outbreak. Prophylactic immunoglobulin would confer what type of immunity to the care workers and noninfected children?

a. Passive immunity

b. Active immunity

c. Herd immunity

d. Nonspecific immunity

A

Rationale

a. Passive immunity is the transfer of antibody produced by one human (or other animal) to another. Passive immunity provides protection against some infections, but this protection is temporary. The antibodies will degrade during a period of weeks to months, and the recipient will no longer be protected.

How well did you know this?
1
Not at all
2
3
4
5
Perfectly
15
Q

You receive a preliminary microbiology report from a blood culture that notes the presence of a few alpha-hemolytic Streptococcus spp. Which of the following organisms is most likely to be identified in the final report?

a. Streptococcus agalactiae

b. Streptococcus pyogenes

c. Group G Streptococcus

d. Streptococcus pneumoniae

A

Rationale

d. Streptococcus pneumoniae is an alpha-hemolytic Streptococcus. Streptococcus pyogenes (group A Streptococcus), Streptococcus agalactiae (group B Streptococcus), and group G Streptococcus are all beta-hemolytic streptococci.

How well did you know this?
1
Not at all
2
3
4
5
Perfectly
16
Q

Why is handwashing preferred over the use of alcohol-based hand sanitizer (ABHS) after caring for a patient with Clostridioides difficile?

a. C. difficile spores are resistant to alcohol.

b. Soap and water are more readily available in all settings than ABHS.

c. ABHS can be irritating to the skin when spores are present.

d. Workers are more compliant with handwashing.

A

Rationale

a. Spores are resistant to alcohol. Handwashing must be used to remove spores since alcohol does not effectively kill C. difficile spores.

How well did you know this?
1
Not at all
2
3
4
5
Perfectly
17
Q

During a recent hurricane, a hospital experiences significant water intrusion impacting all levels of safe operation. Incident command has made the decision to evacuate all patients. There are currently six patients on isolation precautions. To speed the evacuation along, the infection preventionist (IP) was asked to transport as many patients in pairs as possible. Based on the information in the below, what should the IP recommend?

Patient A
Adult with history of methicillin-resistant Staphylococcus aureus (MRSA) admitted for knee replacement; no known active infection.

Patient B
Adult with lower airway infection; ruling out TB, test results are not available.

Patient C
Adult with active vancomycin-resistant enterococci (VRE) infection in a foot wound.

Patient D
Child with chickenpox; most lesions have healed.

Patient E

Adult with C. difficile infection; last diarrheal episode was 18 hours ago.

Patient F
Child with diarrhea of unknown etiology; last episode was 18 hours ago.

a. Cover remaining lesions of patient D and the foot wound of patient C, then transport together.

b. Transport patient E and F together as they are both 18 hours from last diarrheal episode.

c. Cover patient C’s wound and transport with patient A.

d. Cover remaining lesions of patient D and transport with patient F.

A

Rationale

c. Patients B and D both require airborne precautions and should not be transferred with any other patients. Patients with C. difficile infections (CDI) should remain in isolation until a minimum of 48 hours after their last diarrheal episode; as such, they should only be cohorted with other patients with confirmed CDIs.

Patients with contained wounds may be cohorted with other patients with minimal risk. Of the six patients, only patients A and C could be safely transported together.

How well did you know this?
1
Not at all
2
3
4
5
Perfectly
18
Q

An infection preventionist (IP) receives a call from the operating room (OR) that there is a water leak in the sterile storage area. Which of the following actions should the IP recommend OR staff take?

a. Items should be returned to the sterile processing department for complete re-sterilization (clean, wash, wrap, and sterilize).

b. Items should be sent out to another company for re-sterilization.

c. The outer wrap should be removed and re-sterilized.

d. Items should be left to air dry before next use.

A

Rationale

a. Sterile supplies can no longer be considered sterile if there is water contamination in the storage area. The Centers for Disease Control and Prevention (CDC) and the Association for the Advancement of Medical Instrumentation (AAMI) recommend that such items be re-sterilized.

How well did you know this?
1
Not at all
2
3
4
5
Perfectly
19
Q

What is the purpose of a surveillance program?

a. To provide overall data to guide the quality improvement department’s efforts

b. To provide actionable data by identifying all potential risks.

c. To provide actionable data by identifying all unknown and known risks

d. To provide actionable data by identifying the most important risks

A

Rationale

b. An effective surveillance program should include information for detecting outbreaks and instituting control measures to prevent disease transmission based on all potential risks that might be encountered within a given healthcare setting.

How well did you know this?
1
Not at all
2
3
4
5
Perfectly
20
Q

Cavitation is a term used to describe the cleaning process used during which of the following procedures?

a. Pasteurization

b. High-level disinfection

c. Cleaning by ultrasonic washer

d. Cleaning by washer-disinfector

A

Rationale

c. Ultrasonic energy is an effective technology routinely used to clean surgical and dental instruments prior to terminal sterilization. Ultrasonic cleaning uses cavitation bubbles induced by high frequency pressure (sound) waves to agitate a liquid. The agitation produces high forces on contaminants adhering to substrates like metals, plastics, glass, rubber, and ceramics. This action also penetrates blind holes, cracks, and recesses.

How well did you know this?
1
Not at all
2
3
4
5
Perfectly
21
Q

What percentage of data will fall within two standard deviations of the mean in a normal distribution?

a. 75%

b. 99%

c. 68%

d. 95%

A

Rationale

d. In a normal distribution, 95% of data fall within two standard deviations of the mean.

How well did you know this?
1
Not at all
2
3
4
5
Perfectly
22
Q

**What is the central line device utilization ratio for a facility that has 200 central line days and 8,000 patient days?

a. 0.025

b. 2.5

c. 0.25

d. 0.05

A

Rationale

a. The device utilization ratio allows the infection preventionist to monitor outcomes or processes; it is given as a percent by dividing the patient days by the number of device days. In the above example, the device utilization ratio is 200 / 8000 = 0.025, or 2.5%.

How well did you know this?
1
Not at all
2
3
4
5
Perfectly
23
Q

How should drying be facilitated to reduce microbial contamination and proliferation in endoscopes?

a. Blow dry with compressed air, rinse with tap water, and hang vertically to dry.

b. Blow compressed air through the channel and rinse with 70% ethyl or isopropyl alcohol.

c. Rinse with tap water and blow compressed air through the channels.

d. Rinse with alcohol, hang vertically to dry, and store in a case to keep clean.

A

Rationale

b. Endoscope disinfection or sterilization with a liquid chemical sterilant involves five steps after leak testing:

  1. Clean mechanically all internal and external surfaces, including brushing internal channels and flushing each internal channel with water and a detergent or enzymatic cleaners (leak testing is recommended for endoscopes before immersion).
  2. Disinfect by immersing endoscope in high-level disinfectant (or chemical sterilant) and perfusing the disinfectant (to eliminate air pockets and ensure contact of the germicide with the internal channels) into all accessible channels—such as the suction/biopsy and air/water channels—and then expose for the product-specific recommended time.
  3. Rinse the endoscope and all channels with sterile water, filtered water (commonly used with automated endoscope reprocessors [AERs]), or tap water (i.e., high-quality potable water that meets federal clean water standards at the point of use).
  4. Flush the insertion tube and inner channels with alcohol.
  5. Dry with forced air before storage.
How well did you know this?
1
Not at all
2
3
4
5
Perfectly
24
Q

Which of the following statements describes a disadvantage of polymerase chain reaction (PCR) testing?

a. Antibiotic susceptibility testing cannot be performed.

b. It cannot be used after antibiotics have been administered.

c. Low numbers of organisms cannot be detected.

d. Dead organisms cannot be detected.

A

Rationale

a. Antibiotic testing cannot be performed using PCR. The other statements listed above describe advantages of PCR testing: It can be used to identify organisms that are dead, present in low numbers, or have been treated with antibiotics.

How well did you know this?
1
Not at all
2
3
4
5
Perfectly
25
Q

Early-onset neonatal sepsis is most likely caused by which of the following organisms?

a. Bacteroides fragilis

b. Candida spp.

c. Group B streptococcus

d. N. gonorrhea

A

Rationale

c. Early-onset neonatal sepsis is usually caused by organisms acquired intrapartum, and symptoms appear within six hours of birth. Most cases are caused by Group B streptococcus and gram-negative enteric organisms (predominantly E. coli).

How well did you know this?
1
Not at all
2
3
4
5
Perfectly
26
Q

What is the first thing that an infection preventionist (IP) should do when a sterilizer issue is identified?

a. Contact risk management.

b. Unwrap then rewrap instruments.

c. Take the sterilizer out of service.

d. Inspect instruments to see whether they need to be recleaned.

A

Rationale

c. It is vital that problematic sterilizers be removed from service to prevent any further risk of contaminated instruments being used.

How well did you know this?
1
Not at all
2
3
4
5
Perfectly
27
Q

Which of the following work restrictions would be required for a healthcare worker (HCW) with a confirmed case of herpes simplex on their hands (herpetic whitlow)?

a. HCW should be excluded from patient contact and the patient’s environment until their lesions heal.

b. HCW can work in the patient’s environment but should be excluded from patient contact until their lesions heal.

c. HCW should not be restricted from work.

d. HCW should cover their herpetic lesions while giving patient care.

A

Rationale

a. Herpetic whitlow can be spread by patient contact or environmental contamination. As such, infected HCWs should be excluded from patient contact and the patient’s environment until their lesions heal.

How well did you know this?
1
Not at all
2
3
4
5
Perfectly
28
Q

When should an infection prevention and control (IPC) risk assessment be updated?

a. When leadership requires it

b. Before an announced visit by an accreditation organization

c. When new risks arise or already identified risks change

d. When a complaint is filed against the facility

A

Rationale

c. Risk assessment is an ongoing part of an infection prevention and control program (IPCP). The risk assessment should be updated whenever new risks arise (e.g., new procedures, changes in patient population, or a pandemic), or when there are changes to already identified risks.

How well did you know this?
1
Not at all
2
3
4
5
Perfectly
29
Q

The value analysis committee is reviewing a new product for cleaning equipment at the point of use. Which of the following product factors is most important to consider as it relates to infection prevention and control (IPC)?

a. Storage

b. Contact time

c. Cost

d. Disposal

A

Rationale

b. Although all options represent important factors in the consideration of a new cleaning product, the main IPC concern is staff compliance: shorter contact times are associated with higher cleaning compliance rates.

How well did you know this?
1
Not at all
2
3
4
5
Perfectly
30
Q

Which of the following methods is preferred when point-of-use cleaning is required?

a. Ultrasonic cleaner

b. Washer-decontaminator

c. Ultraviolet (UV) light disinfection

d. Manual cleaning

A

Rationale

d. Some equipment—such as endoscopes and surgical instruments—are fragile or difficult to clean. For these reasons, mechanical cleaning methods are not feasible and point-of-use cleaning should be done instead; manual cleaning is preferred method for point-of-use cleaning. Point-of-use cleaning also prevents the formation of biofilms.

How well did you know this?
1
Not at all
2
3
4
5
Perfectly
31
Q

What kind of gene acquisition occurs when environmental deoxyribonucleic acid (DNA) enters a bacterial cell?

a. Transduction

b. Replication

c. Conjugation

d. Transformation

A

Rationale

d. Transformation occurs when naked DNA from the environment enters a bacterium. Conjugation is when all or part of plasmid is transferred from donor to recipient cell. Transduction occurs when DNA is transferred from one bacterial cell to another via a virus capable of infecting bacteria.

How well did you know this?
1
Not at all
2
3
4
5
Perfectly
32
Q

A patient colonized with vancomycin-resistant enterococci (VRE) is receiving hemodialysis on an outpatient basis. Which of the following statements is true about this patient?

a.They cannot be cohorted with known VRE patients in a specific area of the dialysis unit.

b. They have a lower risk of bacteremia then other patients with central catheters.

c. They should be observed for pyrogenic reactions from 24–48 hours following hemodialysis.

d. They have the potential for fecal contamination of the environment.

A

**Rationale **

d. Patients with VRE may be cohorted. Although many patients become colonized by patient-to-patient spread, environmental contamination does inevitably occur; and significant reservoirs may, therefore, exist and persist in clinical care areas, such as a dialysis unit.

How well did you know this?
1
Not at all
2
3
4
5
Perfectly
33
Q

A recent study demonstrated that more than 40 percent of healthcare personnel (HCP) have come to work with flu-like illnesses. Which of the following interventions is most effective at thwarting presenteeism?

a. Implementing mandatory exclusion rules

b. Requiring a medical certificate for absences

c. Restricting paid sick leave

d. Removing systematic processes for screening employees

A

Rationale

a. Presenteeism occurs when employees go to work despite having a medical illness that prevents them from fully fulfilling their job duties. Policies to prevent presenteeism should include the availability of unrestricted paid sick leave, systematic processes for screening ill employees, and mandatory exclusion rules.

How well did you know this?
1
Not at all
2
3
4
5
Perfectly
34
Q

Which of the following measures can help an infection preventionist (IP) to identify whether or not changes to facility practices are leading to intended improvements?

a. Risk measures

b. Strategic measures

c. Outcome measures

d. Process measures

A

Rationale

c. Outcome measures indicate whether changes are actually leading to improvement—that is, helping to achieve the overall aim of preventing healthcare-associated infections (HAIs); examples include the rate of occurrence of methicillin‐resistant Staphylococcus aureus (MRSA) per 1,000 patient days and the percent of patients with C. difficile-associated diseases.

How well did you know this?
1
Not at all
2
3
4
5
Perfectly
35
Q

A data set is comprised of six values: 7, 12, 6, 9, 15, and 11. What is the median for this data set?

a. 11

b. 10

c. 9

d. 13

A

Rationale

b. The terms mean, median, mode, and range describe properties of statistical distributions. The median is the middle number in a sorted list of numbers; it depends on whether the number of data points is odd or even—if the former, then the median is the value of the middle term; if the latter, then it is the average of the two middle terms.

To calculate the median in this example, the data set is first sorted (6, 7, 9, 11, 12, 15). As there is an even number of terms, the two middle numbers (9 and 11) would be averaged: (9 + 11) / 2 = 10.

How well did you know this?
1
Not at all
2
3
4
5
Perfectly
36
Q

If an infection preventionist (IP) helps learners to organize themselves so that they share the responsibility for mutual inquiry, which of the following conditions of adult education is met?

a. The environment is characterized by physical comfort and mutual respect.

b. Learners feel the need to learn.

c. Learners participate actively in the learning process.

d. Learners perceive the goals of learning to be their goals.

A

Rationale

c. Active learning is the process of learning via engaging with the content. By implementing active learning strategies, students can interact with the material via activities such as completing a task or engaging with their surroundings (e.g., discussions or debates) in ways that promote analytical thought.

How well did you know this?
1
Not at all
2
3
4
5
Perfectly
37
Q

The director of the intensive care unit (ICU) asks the infection preventionist (IP) whether ventilator tubing can be reprocessed and reused. Which of the following actions should the IP take?

a. Experiment with different cleaning and disinfection products to identify an appropriate reprocessing method.

b. Defer the question to the respiratory therapy department .

c. Consult with the sterile processing department to determine whether the ventilator tubing can be reprocessed.

d. Review ventilator tubing packaging and the manufacturer’s instructions for use (IFU) to determine whether it is a single-use item or one that can be reprocessed.

A

Rationale

d. Product packaging and manufacturer’s IFU provide information about whether the product is single-use or multiple-use (i.e., can be reprocessed). The IP is the appropriate subject-matter expert to answer this question for the ICU director since it is an infection prevention and control (IPC) responsibility.

How well did you know this?
1
Not at all
2
3
4
5
Perfectly
38
Q

A 50-year-old healthcare worker (HCW) who has never received a pneumococcal conjugate vaccine (PCV) requests information about them. What information about unvaccinated adults receiving PCVs should the IP share with the HCW?

a. That they can receive a single dose of PCV15 followed 12 months later by one dose of PCV23; alternatively, they can receive one dose of PCV20

b. That they should delay receiving any PCV vaccines until they are 65 years old, at which point they can receive any of the available PCVs

c. That they should receive a single dose of PCV23, with no follow-up

d. That they can receive PVC15, PVC20, and PCV23 if they are spaced 12 months apart

A

Rationale

a. The Centers for Disease Control and Prevention (CDC) recommends that adults aged 18–64 receive a single dose of PCV15 followed 12 months later by one dose of PCV23; alternatively, they can receive one dose of PCV20.

How well did you know this?
1
Not at all
2
3
4
5
Perfectly
39
Q

Which of the following is an example of an outcome measure?

a. Rates of healthcare-associated infections (HAIs)

b. Ratio of infectious disease physicians to patients

c. Percentage of patients receiving the pneumococcal vaccine

d. Percentage of patients receiving preoperative antibiotics

A

Rationale

a. HAIs are examples of undesirable events. The ratio of physicians is an example of a structural measure. The percentage of patients with immunizations or preoperative antibiotics are examples of process measures.

How well did you know this?
1
Not at all
2
3
4
5
Perfectly
40
Q

An infection preventionist (IP) is asked to complete an infection control risk assessment (ICRA) for the removal of carpeting outside a patient care area. Which of the following should the IP prioritize highest when assessing this project?

a. Assess the patient care population at risk to determine mitigation strategies.

b. Assess the scope of the overall project.

c. Ensure that the project is done at night.

d. Ensure that the environmental services (EVS) department is informed of cleanup efforts.

A

Rationale

a. Although all options describe important considerations, the assessment of the patient population at risk is the highest priority for the ICRA as it will inform risk mitigation strategies.

How well did you know this?
1
Not at all
2
3
4
5
Perfectly
41
Q

A healthcare worker (HCW) who has not received the hepatitis B (HBV) vaccine has been exposed by needlestick from a patient with known HBV infection. Which of the following post-exposure treatments should the infection preventionist (IP) recommend?

a. Start the HBV vaccine series and administer HBV immune globulin (HBIG) within 24 hours post-exposure.

b. Administer one dose of the HBV vaccine.

c. Post-exposure prophylaxis (PEP) is not recommended.

d. Administer one dose of HBIG within 24 hours post-exposure.

A

Rationale

a. When an unvaccinated HCW is exposed to HBV, they should be administered the HBV vaccine series and HBIG, preferably within 24 hours post-exposure.

How well did you know this?
1
Not at all
2
3
4
5
Perfectly
42
Q

What is the required first step in surgical instrument reprocessing?

a. Low-level disinfection

b. High-level disinfection

c. Immediate-use sterilization

d. Cleaning

A

Rationale

d. All surgical instruments should be cleaned at the point of use to remove any organic matter. The instruments must then be sterilized. Low- and high-level disinfection are not applicable to surgical instruments.

How well did you know this?
1
Not at all
2
3
4
5
Perfectly
43
Q

According to Centers for Disease Control and Prevention (CDC) guidelines, which of the following statements regarding the prevention of surgical site infections (SSIs) is true?

a. Apply antimicrobial agents (e.g., ointments, solutions, or powders) to the surgical incision for the prevention of SSIs.

b. Remove hair at the operative site because its presence might interfere with the surgical operation.

c. Administer preoperative antimicrobial agents only when indicated, based on published clinical practice guidelines and timed such that a bactericidal concentration of the agents is established in the serum and tissues after the incision is made.

d. Implement perioperative glycemic control and use blood glucose target levels of less than 200 mg/dL in patients both with and without diabetes.

A

Rationale

d. Preoperative antimicrobial agents should only be administered when indicated, based on published clinical practice guidelines and timed such that a bactericidal concentration of the agents is established in the serum and tissues when the incision is made. Antimicrobial agents (e.g., ointments, solutions, or powders) should not be applied to the surgical incision for the prevention of SSIs, and hair should not be removed at the operative site unless its presence will interfere with the operation.

How well did you know this?
1
Not at all
2
3
4
5
Perfectly
44
Q

An adenovirus outbreak has been identified in a neonatal intensive care unit (NICU). Epidemiological investigation reveals 18 primary cases among the 22 neonates housed on the unit, 11 secondary cases among the 20 employees who were potentially exposed, and 6 secondary cases among 15 parents who had exposure. What is the secondary attack rate among exposed contacts?

a. 48.6%

b. 55.2%

c. 74.3%

d. 40.5%

A

Rationale

a. A secondary attack rate represents the probability that an infection occurs among susceptible people within a specific group (e.g., household or close contacts). It is calculated as the number of new cases among contact during the specified time divided by the total number of contacts at risk, multiplied by 100. In this example: ([11 + 6] / [20 + 15]) x 100 = 48.6%.

How well did you know this?
1
Not at all
2
3
4
5
Perfectly
45
Q

While performing rounds, an infection preventionist (IP) notices a healthcare worker (HCW) wiping down an endocavitary probe with a low-level disinfectant wipe. What corrective action or feedback should the IP provide to this HCW?

a. Collaborate with the HCW in cleaning the endocavitary probe and explain Bloom’s taxonomy.

b. Explain the Spaulding classification model and review manufacturer’s instructions for use (IFU).

c. Remind the HCW that endocavitary probes should only be sterilized, not disinfected.

d. Do nothing: endocavitary probes are noncritical items and the use of a low-level disinfectant is acceptable.

A

Rationale

b. Before using semi-critical devices, HCWs must understand the Spaulding classification model and be aware of any manufacturer’s IFU. Sterilization should only be used if recommended by the manufacturer IFU. Bloom’s taxonomy concerns learning behaviors, not cleaning and disinfection protocols.

How well did you know this?
1
Not at all
2
3
4
5
Perfectly
46
Q

Statistical tests are used to describe data from research studies. The specific tests used in a study to analyze this data are typically presented in what section of a research paper?

a. Introduction

b. Methods

c. Results

d. Discussion

A

Rationale

b. The discussion section fits the results of a study into the framework of what is known about the problem and then compares those results to other studies. The introduction of a research study will describe the study’s purpose and rationale. The methods section details the tools and procedures, including statistical tests, used in the study.

How well did you know this?
1
Not at all
2
3
4
5
Perfectly
47
Q

An environmental services (EVS) worker vaccinated against hepatitis B (HBV) sustains a needlestick injury while emptying trash. The source patient cannot be identified. Which of the following actions should the infection preventionist (IP) recommended be taken within 72 hours of the needlestick?

a. Provide hepatitis A (HAV) vaccine.

b. Test the worker to establish continued HBV immunity.

c. Perform a baseline human immunodeficiency virus (HIV) test.

d. Provide hepatitis C (HCV) vaccine.

A

Rationale

c. Post-exposure baseline HIV testing is recommended when the source patient is known to be HIV positive or when the source patient’s HIV status cannot be determined. Workers who have demonstrated HBV immunity after vaccination do not need retesting. There is no HCV vaccine. Testing for HAV, a foodborne pathogen, is not required after needlestick exposures.

How well did you know this?
1
Not at all
2
3
4
5
Perfectly
48
Q

An infection preventionist (IP) identifies a trend of norovirus cases presenting to the emergency department (ED). Who should follow up with the community to perform contact tracing and programs for prevention?

a. IP

b. Public health officials

c. ED personnel

d. Chief nursing officer

A

Rationale

b. The IP performs surveillance within their facility, but they should notify the public health department of any identified trends. Public health officials are then responsible for using the IP’s surveillance findings to prevent community-related infections and trends and to enact public health policies.

How well did you know this?
1
Not at all
2
3
4
5
Perfectly
49
Q

A patient presents with mild respiratory allergies and disseminated skin lesions; the presiding physician starts treatment with the antiviral acyclovir. The infection preventionist (IP) is consulted. What precautions should they recommend?

a. Airborne and contact

b. Airborne

c. Contact

d. Droplet and contact

A

Rationale

a. Shingles (herpes zoster) is a manifestation of a recurrent infection; disseminated presentation requires airborne and contact precautions.

How well did you know this?
1
Not at all
2
3
4
5
Perfectly
50
Q

An infection preventionist (IP) is making rounds with nursing representatives to assess compliance with cleaning and disinfection protocols for portable equipment. They observe the use of intravenous (IV) pumps with evidence of surface damage (corrosion). Which of the following steps should the IP take first?

a. Conclude that the corrosion is due to the age of the equipment.

b. Check the expiration date of the disinfecting wipes.

c. Review the IV pump manufacturer’s instructions for use (IFU).

d. Remove the product from use immediately.

A

Rationale

c. The IV pump manufacturer’s IFU will provide directions for how to clean and disinfect the machine, including the type of products that should be used. If the equipment is not being cleaned and disinfected according to its IFU, the corrosion may be caused by an inappropriate cleaning or disinfecting product. It is not feasible to remove in-use equipment immediately. The expiration date of the disinfecting wipes has no bearing the presence of the corrosion.

How well did you know this?
1
Not at all
2
3
4
5
Perfectly
51
Q

Which of the following types of vaccines should be avoided by pregnant healthcare personnel (HCP)?

a. Inactivated vaccines

b. Live-virus vaccines

c. Coronavirus disease 2019 (COVID-19) vaccines

d. All vaccines

A

Rationale

b. Live-virus vaccines are contraindicated for pregnant women. The other types of vaccines listed are all considered safe during pregnancy.

How well did you know this?
1
Not at all
2
3
4
5
Perfectly
52
Q

A patient presenting with pneumonia has a respiratory culture that grows Escherichia coli; susceptibility testing shows resistance to amoxicillin, ceftazidime, ceftriaxone, and ciprofloxacin and sensitivity to amoxicillin-clavulanate, piperacillin-tazobactam, and gentamicin. What precautions should the infection preventionist (IP) recommend for this patient?

a. Droplet

b. Contact

c. Airborne

d. Standard

A

Rationale

b. The patient is infected with an organism (Escherichia coli) that exhibits extended spectrum beta-lactamase (EBSL), which hydrolyses late generation cephalosporins and penicillins and can be inhibited by clavulanate and tazobactam. The recommended type of precautions for ESBLs is contact.

How well did you know this?
1
Not at all
2
3
4
5
Perfectly
53
Q

Which of the following organisms requires a living host to grow and reproduce?

a. Bacteria

b. Viruses

c. Prions

d. Fungi

A

Rationale

b. Viruses reproduce through the spread of virions, which are inert and do not grow or multiply until they enter a living cell. Virions are taken in by living host cells, where they modify the cell metabolism to produce a new nucleic acid and protein coat. Once the particles are assembled and released by the cell they can invade other host cells.

How well did you know this?
1
Not at all
2
3
4
5
Perfectly
54
Q

What do the World Health Organization (WHO) and Centers for Disease Control and Prevention (CDC) provide to healthcare facilities?

a. Best practice guidelines based on the most up-to-date scientific evidence

b. Requirements that must be followed under penalty of law

c. Authority to follow a regulatory agency

d. Regulations enforceable by legal penalties

A

Rationale

a. The WHO and CDC do not set laws or regulations. Instead, they provide guidelines based on recognized expert consensus.

How well did you know this?
1
Not at all
2
3
4
5
Perfectly
55
Q

During an infection prevention and control (ICP) committee meeting, it is mentioned that the hospital has a low patient influenza vaccination rate and omissions in assessing new admissions for influenza vaccination prior to hospital admission. Which of the following strategies should the infection preventionist (IP) recommend to improve the immunization rate?

a. Encouraging patients to discuss vaccines with their primary care physician

b. Offering all patients vaccine information statements (VISs) upon admission

c. Encouraging nurses to ask patients about their vaccination status upon admission

d. Standing orders for vaccines

A

Rationale

d. Standing orders for vaccines have been shown to significantly improve the rate of vaccination.

How well did you know this?
1
Not at all
2
3
4
5
Perfectly
56
Q

An infection preventionist (IP) should immediately notify public health authorities after receiving which of the following laboratory reports?

a. Four cases of gram-negative bacteria in urine

b. Two cases of Clostridioides difficile from dialysis patients

c. One case of probable Bacillus anthracis from the blood culture of a prominent politician

d. Three cases of Staphylococcus epidermidis in blood cultures

A

Rationale

c. Bacillus anthracis is the causative organism of anthrax. Anthrax is rare in the US and, when detected, should be reported immediately to public health officials to ascertain whether it is related to bioterrorism.

How well did you know this?
1
Not at all
2
3
4
5
Perfectly
57
Q

When making rounds, an infection preventionist (IP) notices that a provider did not perform proper hand hygiene (HH) before quickly stepping out of a patient’s room to answer an urgent call. Such a mistake is known as which of the following types of failure?

a. Skill-based

b. Personal-based

c. Rule-based

d. Knowledge-based

A

Rationale

a. A skill-based failure is any inadvertent mistake made when conducting a routine or automatic task—as, for example, one due to distraction.

How well did you know this?
1
Not at all
2
3
4
5
Perfectly
58
Q

Which of the following items are considered “critical” according to the Spaulding classification?

a. Vascular access devices (VADs), gastrointestinal endoscopes, and laryngoscopes

b. VADs, bedpans, and biopsy probes

c. Bronchoscopes, gastrointestinal scopes, and stethoscopes

d. Blood pressure cuffs, computers, and floors

A

Rationale

a. According to the Spaulding classification, critical items are those that enter sterile sites, such as VADs, gastrointestinal endoscopes, and laryngoscopes. Semi-critical items enter non-sterile sites or contact mucous membranes. Noncritical items come into contact only with intact skin.

How well did you know this?
1
Not at all
2
3
4
5
Perfectly
59
Q

Which of the following learning objectives is based on the cognitive domain?

a. Perform an assessment of the integrity of a central line dressings.

b. Describe the impact of coronavirus disease 2019 (COVID-19) on their practice.

c. Demonstrate the steps of a urinary catheter care bundle.

d. Identify three risk factors for developing a Clostridioides difficile
infection (CDI).

A

Rationale

d. The three domains of learning are cognitive (thinking/head), affective (feelings/emotions), and psychomotor (physical). The cognitive learning domain involves the intellect—the understanding of information and how that develops through application on a scale that increases from basic recall to complex evaluation and creation. Identifying risk factors is an example of knowledge application.

How well did you know this?
1
Not at all
2
3
4
5
Perfectly
60
Q

A patient presents to the emergency department (ED) with diarrhea, nausea, and vomiting. The patient reported eating a premade ham sandwich from a convenience store approximately 10 hours prior. Which of the following organisms is the probable cause?

a. Salmonella enteritidis

b. Shigella sonnei

c. Campylobacter jejuni

d. Listeria monocytogenes

A

Rationale

d. Ready-to-eat deli meats are a common source of Listeria, and the symptoms of Listeria typically appear 9–48 hours after eating contaminated food. Although Salmonella, Shigella, and Campylobacter all cause similar symptoms, they are not commonly associated with eating ready-to-eat deli meats and their symptoms typically occur 1–2 days after eating contaminated food.

How well did you know this?
1
Not at all
2
3
4
5
Perfectly
61
Q

Which of the following statements should an infection preventionist (IP) consider when developing a policy to prevent the transmission of vancomycin-resistant enterococci (VRE)?

a. Person-to-person spread has not been proven.

b. Barrier precautions are ineffective.

c. Environmental contamination plays a role in transmission.

d. VRE is more infectious than vancomycin-sensitive enterococci.

A

Rationale

c. VRE can spread directly to people who contact VRE-contaminated surfaces. VRE is not spread through the air by coughing or sneezing.

62
Q

An infection preventionist (IP) is collaborating with the infection prevention and control (IPC) committee to select clinically important core measures to evaluate program success. What are the four criteria for good accountability measures?

a. Research, process, balancing, adverse effects.

b. Outcome, process, balancing, clinical

c. Mortality, morbidity, infection, devices

d. Research, proximity, accuracy, adverse effects

A

Rationale

d. According to The Joint Commission (TJC), the process for selecting good core measures for IPC includes four accountability measures: research, proximity, accuracy, and adverse effects.

63
Q

Among the following microbes, which is the least resistant to sterilization?

a. Poliovirus

b. Cryptosporidium parvum

c. Human immunodeficiency virus (HIV)

d. Mycobacterium tuberculosis

A

Rationale

c. Microbes have differing resistance to sterilization based on their physical and chemical properties. Vegetative bacteria (such as Staphylococcus aureus) and lipid or medium-sized viruses (such as HIV) are the least resistant of all microbes to sterilization.

64
Q

Which of the following activities can help in identify risks and gaps in an infection prevention and control program (IPCP) as it relates to patient safety?

a. Instituting performance improvement projects

b. Analyzing time data

c. Performing a risk assessment

d. Analyzing process data

A

Rationale

c. A risk assessment can aid the infection preventionist (IP) in identifying potential (and actual) risks and gaps in an IPCP, particularly in regards to patient safety.

65
Q

The director of environmental services (EVS) has developed a validation tool to assess—after staff have been educated on and provided with an appropriate cleaning protocol—the cleaning of high-touch surfaces in patient rooms. What type of performance measure has the EVS director implemented?

a. Structural

b. Process

c. Outcome

d. Clinical

A

Rationale

b. The EVS director has developed a process measure. Process measures can be used to assess the effectiveness of educational programs and compliance with written protocols.

66
Q

Which of the following items is considered “semi-critical” according to the Spaulding classification?

a. Blood pressure cuff

b. Central line

c. Bronchoscope

d. Prosthetic joint implant

A

Rationale

c. According to the Spaulding classification, semi-critical items are those that enter non-sterile sites or contact mucous membranes, such as a bronchoscope. A blood pressure cuff is an example of a noncritical item, and central lines and prosthetic joints are examples of critical items because they enter sterile sites.

67
Q

The epidemic curve depicted in the above chart is an example of which of the following modes of transmission?

a. Propagated

b. Vector-borne

c. Common-source

d. Person-to-person

A

Rationale

c. A common-source outbreak is defined by the CDC as “an outbreak in which a group of persons are all exposed to an infectious agent or a toxin from the same source.” In some common-source outbreaks, case-patients may have been exposed over a period of days or weeks (or longer), as was the case in the outbreak shown above.

68
Q

The most valid and reliable results from research studies are typically found in which of the following sources?

a. Peer-reviewed journals

b. Product brochures

c. Manufacturer white papers and background briefings

d. Webinars and online briefings

A

Rationale

a. Peer-reviewed journals are scrutinized by editorial staff and experts in the field before being published. Product brochures and materials provided by manufacturers are designed to promote the unique features of their products. Webinar and online briefings may or may not be reviewed for methods or content.

69
Q

While reviewing microbiology test results, an infection preventionist (IP) notices a positive culture for norovirus. They report this information to the local department of health. The following day, the IP learns that another patient on the same unit has tested positive for norovirus. Nurses also report new-onset vomiting and diarrhea for three additional patients on the same unit and two healthcare providers. In light of this information, which of the following actions should the IP take?

a. Notify the local department of health and the facility’s emergency management team.

b. Notify the local department of health and conduct an outbreak investigation.

c. Notify the Centers for Disease Control and Prevention (CDC) and close the unit.

d. Notify the World Health Organization (WHO) and conduct an outbreak investigation.

A

Rationale

b. All outbreaks should be reported to the local health department because public health officials can consult on outbreak investigations and provide assistance or support—including laboratory services if needed—during an actual outbreak.

70
Q

Which of the following processes eliminates many or all pathogenic microorganisms (except bacterial spores) on inanimate objects?

a. Sterilization

b. Disinfection

c. Cleaning

d. Pre-cleaning

A

Rationale

b. Disinfection processes involve the use of liquid chemicals or wet pasteurization. It does not kill bacterial spores, but it does kill all other pathogenic organisms.

71
Q

A patient scheduled for elective colon surgery was given cefazolin and metronidazole within an hour prior to incision. What type of antimicrobial use is this?

a. Prophylactic

b. Diagnostic

c. Empiric

d. Therapeutic

A

Rationale

a. Antimicrobial prophylaxis is the use of antimicrobials to prevent infection prior to surgery or other procedure. Empiric use is directed at active infections before information on the causative pathogen is available. Therapeutic use is pathogen directed. Diagnostic use targets known diseases.

72
Q

An infection preventionist (IP) is performing rounds on the patient care unit and notices an alcove that is overflowing with equipment, waste containers, and food carts. It is difficult to determine what items are clean and what items are dirty. Which of the following steps should the IP take next?

a. Go back to the office and draft an email to the unit supervisor.

b. Make a phone call to the house supervisor and ask them to address it.

c. Do nothing but follow up in one week to see whether organization is better.

d. Speak with the team and provide just-in-time education.

A

Rationale

d. When monitoring and evaluating processes or procedures, the IP should use direct observation and provide immediate feedback to the frontline team. Drafting an email or making a phone call shifts the responsibility to others. Doing nothing could potentially lead to cross-contamination issues.

73
Q

In order to decrease hospital-associated infections (HAIs) within their healthcare facility, an IP has established a process for providing feedback and education to healthcare personnel (HCP) for observed lapses in practices. Which of the following would be an effective means of ensuring that this process is widely accepted among staff?

a. Emergency management processes

b. Catheter-associated urinary tract infection (CAUTI), central line-associated bloodstream infection (CLABSI), and C. difficile infection (CDI) prevention champions

c. Identification of immunocompromised patients

d. Organizational level of preparedness

A

Rationale

b. Prevention champions are respected individuals with strong communication skills who are knowledgeable and enthusiastic about the infection prevention and control (IPC). They help to provide feedback and education to HCP when lapses in practices are observed.

74
Q

Colonization is the asymptomatic presence of microorganisms. Some synonyms for colonization are “normal flora” and “asymptomatic carriage.” Which of the following terms is also used to indicate colonization?

a. Commensal

b. Opportunist

c. Contaminant

d. Abscess

A

Rationale

a. The word commensal, like the word colonization, indicates that microorganisms are present in a host without causing symptoms of disease.

75
Q

The infection prevention and control program (IPCP) is integral to organizational quality assurance. What is quality assurance?

a. A tool used to seek root causes and implement performance improvement, incorporating information from lessons learned

b. The use of a quantitative tool that provides an indication of performance

c. A proactive, ongoing cycle that focuses on continuous improvement of overall patient outcomes, processes, and systems

d. A process for achieving quality standards and meeting a level of acceptable patient care that focuses on identifying outliers; regulatory guidelines establish thresholds

A

Rationale

d. Quality assurance is a process for achieving quality standards and meeting a level of acceptable patient care. Performance improvement is a proactive, ongoing cycle that focuses on continuous improvement of outcomes, processes, and systems. Both are necessary components of a quality assurance and performance improvement (QAPI) program. Root cause analysis (RCA) is a tool used to seek root causes. A quantitative tool that provides an indication of performance is known as a performance measure.

76
Q

What sterile processing department documentation should an infection preventionist (IP) use to assess the effectiveness of sterilization cycles?

a. Records of equipment maintenance

b. Documentation of sterile supply rotation

c. Number of completed sterilization cycles

d. Exposure time and temperature and the results of biological indicator (BI) testing

A

Rationale

d. Sterilizer exposure time and temperature, together with the BI testing (e.g., a Bowie–Dick test), should be documented to assist the IP in verifying effective sterilization cycles. The other options provide information that may be useful to know or identify records that should be kept, but they do not directly help to assess the effectiveness of sterilization cycles.

77
Q

Tools used to diagnose pulmonary tuberculosis (TB) include:

  • Patient reports of cough and night sweats.
  • Microbiology tests showing acid-fast bacilli (AFB).
  • Tuberculin skin tests (TSTs) or interferon-gamma release assays (IGRAs) to detect immune reaction to Mycobacterium tuberculosis.

Which of the following is another important tool for diagnosing TB?

a. Fluid intake and output measurements

b. Imaging of the chest through x-ray or computerized tomography (CT)-scan

c. Blood and urine cultures

d. Blood glucose measurement

A

Rationale

b. Chest x-rays and CT-scans are useful for showing pathology closely associated with TB. Some common findings are cavities, consolidations, nodules, or evidence of disseminated infection. While other indicators of failing health may be present in certain patients, such indicators are not specific to TB.

78
Q

An infection preventionist (IP) has identified an employee with a latent tuberculosis infection (LTBI) who works in the unit where negative air pressure rooms house patients with active tuberculosis (TB) disease. However, this employee has not cared for any TB patients in the past year. They have not been exposed to any known persons with TB either. Which of the following statements describes the likeliest reason the employee has become infected?

a. The TB test is incorrect.

b. The isolation rooms have only 10 air changes per hour (ACH)

c. The isolation rooms are close to the nursing station.

d. The heating, ventilation, and air condition (HVAC) system malfunctioned at some point during the last year.

A

Rationale

d. The detection and identification of certain healthcare-associated infections (HAIs)—such as healthcare-associated TB—may suggest an HVAC system malfunction. The other options are unlikely to be the cause of the employee testing positive for LTBI.

79
Q

Rehabilitation staff members contacted the facility’s infection preventionist (IP) about numerous cases of vomiting and diarrhea on their unit. The IP collected concurrent data and determined that patients developed symptoms between 12–48 hours (about two days) after contact with the index case and their symptoms resolved within one to two days. What should the IP do to identify additional outbreak cases?

a. Consistently apply a case definition.

b. Consistently review Loeb criteria.

c. Monitor metabolic panel laboratory results.

d. Perform polymerase chain reaction (PCR) testing for Clostridioides difficile

A

Rationale

a. A case definition is a standard set of clinical criteria that an IP can use to identify additional outbreak cases. In the above example, these criteria will include episodes of vomiting, frequency and consistency of diarrhea, patient location, and a defined time period.

80
Q

Which of the following is a disadvantage of ortho-phthalaldehyde (OPA) compared to glutaraldehyde?

a. OPA is less stable than glutaraldehyde.

b. OPA stains skin and clothing.

c. OPA can irritate the eyes and nose.

d. OPA requires activation.

A

Rationale

b. OPA is a high-level disinfectant that has several advantages over glutaraldehyde. OPA does not require activation, does not irritate the eyes and nose, has very good stability, and has better mycobactericidal activity. One disadvantage of OPA over glutaraldehyde, though, is that OPA can stain skin and clothing.

81
Q

A case-control study is designed to examine the risk factors for healthcare-associated bacteriuria. A case is defined as any patient from whom both a urine culture and a blood culture grew the same organism. Which of the following type of patients should an infection preventionist (IP) select as control subjects?

a. Patients with both negative urine and blood culture reports

b. Patients with positive blood and negative urine culture reports

c. Patients with bacteriuria with negative blood culture results

d. Patients with bacteriuria from whom a blood culture was not obtained

A

Rationale

d. To reduce the risk of bias, controls should be selected that are representative of the population being studied. The control group for this example includes patients with bacteremia but minimizes the number of patients with clinical sepsis.

82
Q

An infection preventionist (IP) is reviewing mitigation strategies for a construction project to minimize airborne contamination. Which of the following terms describes what the air pressure should be in the adjacent patient care unit, relative to the construction site?

a. Increased

b. Neutral

c. Laminar

d. Decreased

A

Rationale

a. The Centers for Disease Control and Prevention (CDC) Guidelines for Environmental Infection Control in Healthcare Facilities recommend that, during construction programs, facilities “create and maintain negative air pressure in work zones adjacent to patient care areas and ensure that required engineering controls are maintained.”

83
Q

What is the first step in the reprocessing of endoscopes?

a. Rinse

b. Water soak

c. Clean

d. Disinfect

A

Rationale

c. Cleaning is the first step in the reprocessing of endoscopies. Rinse and disinfect come later. Water soak is not an acceptable component of endoscope reprocessing. Active cleaning methods are needed.

84
Q

The environmental services (EVS) department is responsible for ensuring staff provide a clean environment for patients and staff. What is the infection preventionist (IP) responsible for as it relates to ensuring a clean environment?

a. Choosing the equipment used by EVS to maintain a clean environment

b. Developing the schedule for EVS cleaning and disinfection

c. Participating in terminal cleaning of patient rooms

d. Evaluating cleaning and disinfection policies, procedures, and practices

A

Rationale

d. As an infection prevention and control (IPC) subject-matter expert, the IP is responsible for evaluating cleaning and disinfection policies, procedures, and practices. The other options listed are primarily responsibilities of the EVS department, though the IP may be asked to advise on them.

85
Q

An infection preventionist (IP) consults the exposure control plan for a report of healthcare personnel (HCP) with confirmed cases of Shigella-caused gastroenteritis. Which component of the plan should the IP use to make decisions regarding patient and staff exposures?

a. The post-exposure and rapid response decision-making processes

b. The post-exposure prophylaxis plans

c. The post-exposure education and training requirements

d. The post-exposure source testing process

A

Rationale

a. The post-exposure and rapid response decision-making processes outline the decision tree or algorithm for assessing exposures and associated interventions.

86
Q

The infection prevention and control (IPC) committee is concerned about staffing levels because their program is not meeting regulatory standards. This concern is identified during the annual IPC risk assessment process. What should the infection preventionist (IP) do to address the identified risk?

a. Discuss identified concerns with risk management staff to determine liability risk.

b. Communicate risk assessment findings, rationale, and recommendations to executive leadership.

c. Discuss the dissatisfaction with colleagues and managers to seek advice.

d. Submit a requisition to the human resources department to communicate the need for additional IPC staffing.

A

Rationale

b. Organizational leadership is responsible for ensuring that IPC programs (IPCPs) are adequately resourced. Risk management does not have decision-making rights to increase staffing and neither does the IP. Discussing this concern with colleagues and managers does not result in approval of additional staffing.

87
Q

Which of the following is defined as a small set of evidence‐based interventions for a defined patient population and care setting?

a. Hypothesis

b. Theory

c. System

d. Bundle

A

Rationale

d. Evidence‐based practices are implemented through a variety of patient‐safety tools, but checklists and bundles remain two of the best ways to reduce infection rates and boost compliance among healthcare personnel (HCP), which leads to better patient outcomes.

88
Q

A clinic manager has reported an increasing number of patients who participate in open relationships. Additionally, the infection preventionist (IP) has noted many patients coming into the clinic with painless ulcerations and sores, wart-like growths on their genitals, and penile/vaginal discharge. Some patients have even complained of a rash. Which of the following stakeholders should the IP contact first?

a. Housing authority

b. Chief medical officer

c. House supervisor

d. External public health contacts

A

Rationale

d. External public health contacts should be notified first because the public health department is best suited to assist with potential cluster outbreak management; they can also provide health education regarding the disease processes involved to help prevent further transmission.

89
Q

Your facility has recently been extensively renovated and you are assessing the heating, ventilation, and air conditioning (HVAC) system to ensure that areas with positively pressurized air have not been impacted. Which of the following areas require positive pressure?

a. Soiled linen rooms

b. Triage

c. Bronchoscopy

d. Trauma rooms

A

Rationale

d. Positively pressurized rooms are usually designed to protect a patient, clean supplies, or equipment within the room. Trauma and delivery rooms are examples of rooms in hospitals and outpatient facilities that should be positively pressurized with respect to adjacent areas.

90
Q

Guidelines for preventing surgical site infections (SSIs) have been published by the Centers for Disease Control and Prevention (CDC). The infection preventionist (IP) is using these guidelines to prepare policies and procedures for correctly identifying non-parenteral antimicrobial prophylaxis strategies. Which of the following statements correctly defines a non-parenteral antimicrobial prophylaxis strategy?

a. Maintain perioperative normothermia.

b. Maintain the cleanliness of the operating room (OR) to minimize intraoperative contamination that can lead to infections in implanted devices.

c. Do not apply antimicrobial agents (e.g., ointments, solutions, or powders) to surgical incisions to prevent SSIs.

d. Implement perioperative glycemic control and use blood glucose target levels of less than 200 mg/dL in patients both with and without diabetes.

A

Rationale

c. The prevention of SSIs consists of minimizing the contact of bacteria with the surgical site, neutralizing those bacterial species that do gain access to the wound, reducing adjuvant effects that create a local environment conducive to infection, optimizing the efficiency of host responses to potential pathogens, properly overseeing the environment of care, and managing instruments. The use of antimicrobial agents—such as ointments, solutions, or powders—on surgical incisions is, however, not recommended as a prophylaxis.

91
Q

** A patient newly admitted to a healthcare facility reports having a fever and a maculopapular rash on their palms, soles, and anogenital region. Sometime within the last month, the patient had intimate contact with someone in their social network with similar symptoms. The provider plans on ordering testing to determine the cause. What should the infection preventionist (IP) review in the patient’s chart for to ensure that the correct test is ordered and appropriate isolation measures are in place?**

a. The IP should review whether the patient meets one of the epidemiological criteria for monkeypox. If so, a polymerase chain reaction (PCR) test should be ordered; the patient should be placed on isolation precautions; and staff should wear a gown, gloves, eye protection, and a respirator upon entering the patient’s room.

b. The IP should review whether the patient meets one of the epidemiological criteria for syphilis. If so, an antigen test should be ordered; the patient should be placed on isolation precautions; and staff should wear a gown, gloves, and eye protection upon entering the patient’s room.

c. The IP should review whether the patient meets one of the epidemiological criteria for varicella-zoster virus (VZV). If so, an antigen test should be ordered, but isolation precautions are unnecessary.

d. The IP should review whether the patient meets one of the epidemiological criteria for monkeypox. If so, an antigen test should be ordered, and the patient should be placed on standard precautions (SP).

A

Rationale

a. When a patient is suspected of having monkeypox, the IP should make sure appropriate isolation precautions are in place and maintained. Testing for monkeypox is performed by polymerase chain reaction (PCR) testing or next-generation sequencing of a clinical specimen.

92
Q

A hospital worker says that he prefers to keep his beard, even though his job requires the occasional use of an N95 respirator. He supports his request by stating that Centers for Disease Control and Prevention (CDC) guidelines are only guidelines, not legal requirements, and that adaptations can be made. How should the infection preventionist (IP) respond?

a. The Food and Drug Administration (FDA) has only approved N95 respirators for workers who can achieve an airtight seal between the respirator and the skin.

b. The CDC has regulatory responsibility for airborne diseases.

c. The National Institute for Occupational Safety and Health (NIOSH) requires that he has no facial hair where the N95 respirator has contact with his skin.

d. The Occupational Safety and Health Administration (OSHA) requires N95 respirators to be worn such that facial hair does not interrupt the airtight seal of the respirator.

A

Rationale

d. OSHA’s Respiratory Protection Standard requires that facilities disallow workers from wearing an N95 respirator when an airtight seal between the respirator and the skin cannot be achieved.

93
Q

An infection preventionist (IP) is notified by an intensivist of a patient being treated for sepsis who has been in the intensive care unit (ICU) for over five days. The intensivist notes that the patient had a central line placed upon admission and a recent blood culture was positive for Candida spp. What type of surveillance was performed?

a. Whole-house

b. Multi-disciplinary

c. Active

d. Passive

A

Rationale

d. Passive surveillance relies on those who are not specifically trained in or responsible for surveillance (e.g., healthcare providers and nurses) but who still report healthcare-associated infections (HAIs) to the IP.

94
Q

The engineering department had informed the infection preventionist (IP) that they will be changing the high-efficiency particulate air (HEPA) filters on the air handler in the operating room (OR). This means that the air handler will have to be shut off during this process. Which of the following would be most concerning to the IP?

a. Whether the person changing the filter wears a mask

b. Who will change the filter

c. Whether the OR is cleaned properly while the air handler is turned off

d. Whether the IP is available at work when the air handler is shut off

A

Rationale

c. When an air handler is shut off to an area, dust settles; and so, when the air handler is turned back on, that dust can disperse to other area—causing contamination—unless the area is properly cleaned while the air handler is off, before turning it back on.

95
Q

An infection preventionist (IP) reports flash sterilization data to the infection prevention and control (IPC) committee quarterly. What type of surveillance data is the IP reporting?

a. Time

b. Surveillance

c. Event

d. Demographic

A

Rationale

a. Collecting flash sterilization data and reporting it quarterly allows the IP to analyze data over time to determine if performance improvement activities are effective.

96
Q

The director of nursing asks the infection preventionist (IP) to attend a meeting with the sterile processing department to discuss immediate-use steam sterilization (IUSS) practices. They want to understand the indications for IUSS to determine whether they need to make improvements to this practice. Which of the following steps should the IP take first?

a. Recommend purchase of additional instrumentation.

b. Assess current IUSS rate for the facility.

c. Validate caskets and instruments designed for IUSS according to manufacturer’s instructions for use (IFU).

d. Ensure complete documentation of IUSS cycles.

A

Rationale

b. The IP may end up recommending that additional instrumentation be purchased, and validating manufacturer’s IFU and ensuring that IUSS cycles documentation is complete are both important; however, the IP must first assess the current IUSS rate before they can create plans and objectives for improvement.

97
Q

The manager of a medical nursing unit wants to audit personal protective equipment (PPE) use compliance within their department, but they are unsure of where to start. Which of the following should the infection preventionist (IP) recommend that the manager use to audit compliance?

a. Environmental cultures

b. Passive surveillance

c. Checklists

d. Outbreak investigation tool

A

Rationale

c. Checklists can be easily created to help audit processes within an organization. Understanding how processes are conducted can help generate helpful information for improvement.

98
Q

An infection preventionist (IP) is providing consultation regarding the creation of an employee occupational health program for a new facility. What is the first thing that should be accomplished?

a. Develop a bloodborne pathogens (BBPs) exposure control plan to eliminate or minimize exposures.

b. Provide training to all employees.

c. Create a list of all employees who have experienced exposures.

d. Create a plan to ensure confidentiality of records.

A

Rationale

a. The exposure control plan is the foundation for all activities related to possible exposure; it addresses employee training and confidentiality of records.

99
Q

The interventional radiology department is initiating prostate artery embolization as an alternative to surgical intervention. The procedure requires that a catheter be inserted into an artery in the groin. According to the Spaulding classification, the catheter is considered which of the following types of equipment?

a. Disposable

b. Critical

c. Semi-critical

d. Noncritical

A

Rationale

b. According to the Spaulding classification, the catheter is a critical piece of equipment because it enters a sterile body site. An artery is not a mucous membrane or non-intact skin, for both of which semi-critical equipment is used. Noncritical items only contact intact skin. Disposable equipment is not a level withing the Spaulding classification system.

100
Q

A researcher is assessing the validity of a tool that measures attitudes towards infection prevention and control (IPC) in a sample of infection preventionists (IPs) from acute care hospitals. The association between the tool and the number of evidence-based policies for preventing central-line associated bloodstream infections (CLABSIs) in place was assessed using an analysis of variance. The researcher hypothesized that hospitals with more positive attitudes toward IPC would have more IPC evidence-based policies in place.
Which of the following types of validity is being evaluated?

a. Criterion

b. Content

c. Construct

d. Face

A

Rationale

a. Validity refers to how accurately a method measures what it is intended to measure. Criterion validity (or criterion-related validity) evaluates how accurately a test measures the outcome it was designed to measure—in this case, does the tool accurately measure the attitudes of staff towards IPC?

Face validity refers to whether a scale “appears” to measure what it is supposed to measure. It is the least rigorous method for documenting a test’s validity. Construct validity is the extent to which a measurement tool is truly assessing what it has been designed to assess. Content validity refers to whether a test or scale is measuring all of the components of a given construct.

101
Q

Which of the following core public health concepts or practices is used to detect and contain new diseases?

a. Encouraging staff to follow manufacturer’s instructions for use (IFU)

b. Providing emerging infectious disease oversight and leadership

c. Providing no recommendations for private healthcare organizations

d. Deferring to the United Nations for oversight

A

Rationale

b. The need to contain infectious individuals and quarantine those exposed to disease has led to changes in public health laws and regulatory recommendations, as well as an increased leadership role for public health organizations across the globe. Additionally, the increasing prevalence of an all-hazards approach to emergency preparedness has led to growing alliances between emergency preparedness services, healthcare facilities, and community businesses. Importantly, public health organizations can provide the infrastructure and legal authority that healthcare facilities need to gain the cooperation of the general public to halt or slow the transmission of (and mitigate outcomes associated with) disease in the community at-large.

102
Q

Which of the following statements best describes an infection control risk assessment (ICRA)?

a. A process to mitigate the risk to patients during construction and renovation

b. A process to engage key stakeholders during construction and renovation

c. A process to evaluate water- and air-handling requirements during construction

d. A systematic process that determines the level of risk to patients and defines controls to reduce risk during construction and renovation

A

Rationale

d. Before risk can be mitigated, an assessment of the impacted patient population as well as the level of risk must be determined—this is known as an ICRA, and it is used to assess and define controls for reducing risk during construction and renovation activities. The other options may form part of an ICRA, but they are too narrowly defined.

103
Q

An infection preventionist (IP) has been notified by the local public health department about a cluster of Corynebacterium diphtheriae cases in their community. When assessing unvaccinated individuals, which of the following populations should the IP consider to be at risk?

a. All individuals

b. Individuals undergoing chemotherapy

c. Elderly individuals

d. Pregnant individuals

A

Rationale

a. Corynebacterium diphtheriae is a true pathogen—that is, one that is able to cause infection and illness even in individuals with fully functioning immune systems.

104
Q

An infection preventionist (IP) is called to the emergency department (ED) to consult on a patient with a fever and rash of unknown origin. Which of the following actions should the IP take next?

a. Isolate the patient.

b. Assess the rash.

c. Ask the patient whether they have traveled recently.

d. Request a surveillance skin culture.

A

Rationale

a. Patients identified as potentially infectious should be immediately isolated according to the CDC’s “Identify, Isolate, and Inform” framework.

105
Q

Retrospective data collection methods are appropriate for which of the following?

a. Use of ultraviolet (UV) light for terminal cleaning of isolation rooms on discharge

b. Barrier compliance during construction

c. Compliance with step-by-step patient room cleaning processes

d. Surgical site infection (SSI) monitoring

A

Rationale

d. Retrospective data collection involves the collating of data that have already been collected, with a focus on examining what has already happened. Concurrent surveillance must be used to monitor construction projects and cleaning processes. Data may be recorded on spreadsheets for later review; however, inter-rater reliability is important for valid data.

106
Q

An infection preventionist (IP) is working on an education program to promote the use of incident reports for tracking all infection events and near misses and identifying their root causes. The IP is receiving resistance from staff. What should the IP do to help promote the use of incident reports?

a. Make incident reporting mandatory.

b. Allow staff to report incidents confidentially.

c. Inform staff that incident reports look only at trends in events and do not look at individual errors.

d. Inform staff that incident reports should only be used for events that cause actual harm.

A

Rationale

b. Incident reporting should be voluntary and confidential and it should cover all adverse, sentinel, and near-miss events. The focus should be on how events occur, while underscoring individual accountability and responsibility.

107
Q

Gram stains classify organisms as either gram-positive or -negative. Which of the following cell wall components is the determinant factor in Gram staining classification?

a. Peptidoglycans

b. Lipids

c. Polysaccharides

d. Mycolic acids

A

Rationale

a. Gram staining is fundamental to the phenotypic characterization of bacteria. The staining procedure differentiates organisms of the domain Bacteria according to their cell wall structures. Gram-positive cells have a thick peptidoglycan layer and stain blue to purple. Gram-negative cells have a thin peptidoglycan layer and stain red to pink.

108
Q

An acute care facility is opening an ambulatory surgical center using the latest technology available. The infection preventionist (IP) is asked to write infection prevention and control (IPC) guidelines for a new piece of equipment. Which of the following sources should the IP use as their primary source of information in writing these guidelines?

a. Occupational Safety and Health Administration (OSHA)

b. American College of Physicians (ACP) clinical practice guidelines

c. Association for Professionals in Infection Control and Epidemiology (APIC)

d. The manufacturer’s instructions for use (IFU)

A

Rationale

d. Unless conflicting with local laws, regulations, or medical guidance, manufacturer’s IFU should always be followed for best IPC results and to extend the useful life of equipment.

109
Q

What is an extraction area?

a. Where excess water is removed from textiles after laundering but before conditioning or drying

b. Where textiles are folded

c. Where textiles are either conditioned (partly dried) or fully dried in a dryer or tumbler

d. Where soiled textiles are sorted, usually by textile category and sometimes by degree of soiling or color

A

Rationale

a. An extraction area is where excess water is removed from textiles after laundering but before conditioning or drying. A folding area is where textiles are folded. A receiving area is where soiled textiles are sorted, usually by textile category and sometimes by degree of soiling or color. A conditioning and drying area is where, after extraction, textiles are either conditioned (partly dried) or fully dried in a dryer or tumbler.

110
Q

An infection preventionist (IP) notices that several discharged newborns have been readmitted with staphylococcal infections. Which of the following actions should the IP take first?

a. Gather data on neonatal readmissions to identify post-discharge infections.

b. Observe medical and nursing techniques in the nursery.

c. Obtain surveillance cultures on babies in the nursery.

d. Report this trend at the next meeting of the infection control committee.

A

Rationale

a. One of the first tasks in an outbreak investigation is to verify that a cluster of cases is indeed an outbreak. Some clusters turn out to be true outbreaks with a common cause, others are sporadic and unrelated cases of the same disease, and still others are unrelated cases of similar but unrelated diseases. Early in the investigation process, the IP should look for additional cases to determine the true geographic extent of the problem and the populations it impacts.

111
Q

There is an outbreak of norovirus among workers a local clinic. The IP advises that the clinic be temporarily closed and patients have their appointments rescheduled at nearby clinics. Asymptomatic workers from the impacted clinic are assigned to complete phone-based patient surveys from home. What is this strategy intended to reduce?

a. Burn out

b. Negativity

c. Presenteeism

d. Absenteeism

A

Rationale

c. Presenteeism is the term used for workers reporting to work when ill or contagious to others—often because workers fear losing income when taking days off. Because norovirus symptoms are usually self-reported, can be mild, and require two days off while asymptomatic, workers may be incentivized to report to work while still contagious. A facility can reduce presenteeism by reducing the incentive to come to work ill, such as by allowing workers to complete tasks from home.

112
Q

Which of the following types of environmental monitoring should the infection preventionist (IP) perform during construction or renovation projects?

a. No additional monitoring

b. Visual monitoring

c. Electronic monitoring

d. Air sampling

A

Rationale

b. The visual monitoring of processes established for construction, renovation, or maintenance procedures is an important method for reducing the potential for infection from airborne contamination.

113
Q

Which of the following statements about Centers for Disease Control and Prevention (CDC)-specific hand hygiene (HH) requirements for surgical hand antisepsis is correct?

a. OR personnel should remove any debris from under fingernails using a nail cleaner and scrub brush on a dry surface before operating.

b. OR personnel should remove all rings and bracelets before performing surgical hand antisepsis; only wedding bands are allowed to remain during this process.

c. When performing HH with an alcohol-based hand sanitizer (ABHS), OR personnel should clean their nails on a dry surface, apply the ABHS to their hands and forearms, and then allow them to completely dry before donning sterile gloves.

d. Operating room (OR) personnel should perform HH using soap or a hand sanitizer with persistent antimicrobial activity in accordance with manufacturer’s instructions for use (IFU).

A

Rationale

d. All watches, rings, bracelets, and other hand jewelry (including wedding bands) should be removed before beginning antisepsis. Artificial fingernails or nail extenders must not be worn by any personnel in the OR. Staff should remove any debris from under fingernails using a nail cleaner under running water prior to the first scrub of the day. HH should be performed with soap or a hand sanitizer with persistent antimicrobial activity according to the manufacturer’s IFU; when using an ABHS, hands and forearms should be prewashed with a non-antimicrobial soap and then dried completely before applying the ABHS.

114
Q

The infection prevention and employee health departments are coordinating the annual employee vaccination campaign. The infection preventionist (IP) reviews the temperature log for the storage unit containing the influenza vaccine. The current temperature is 44°F (7°C), the daily minimum was recorded as 44°F (7°C), and the daily maximum was 48°F (9°C). Have the vaccinations been kept within recommended temperature ranges and is any action required?

a. Current and min/max temperatures are both within range. No action is needed.

b. Current temperature is within range; min/max are out of range. Action is required.

c. Current and min/max temperatures are both out of range. Action is required.

d. Current temperature is within range; min/max are out of range. No action is needed.

A

Rationale

b. Vaccines should be protected from light and stored at recommended temperatures. In general, influenza vaccines should be refrigerated at temperatures between 36 and 46°F (2–8°C) and should never be frozen; vaccines that have been frozen should be discarded, and action should be taken if temperatures are out of range.

115
Q

An infection preventionist (IP) is evaluating whether N95 respirators may be effective at reducing the risk of workers getting infected with influenza. The IP presents the question to the infection control committee. Which of the following resources would be best suited to answering this question on a risk assessment?

a. A recent opinion article in national newspaper that advocates for the use of N95 respirators for worker safety during flu season

b. A peer-reviewed journal article covering a four-year randomized clinical trial that concluded that N95 respirators offer no protective benefit against influenza

c. A corporate cost-comparison report that discourages N95 respirators in favor of surgical masks based on a cost analysis

d. An N95 respirator product package insert that lists influenza prevention as an intended use

A

Rationale

b. While performing a risk assessment, it is important that the IP critically appraise relevant literature, especially when there are competing priorities and inconsistent conclusions to the question at hand. Peer-reviewed journal articles are preferable to other publications; randomized clinical trials have greater strength than less rigidly controlled studies. And cost cannot be the primary motivating factor in making infection prevention and control (IPC) decisions.

116
Q

When cleaning patient rooms, which of the following should environmental services (EVS) staff clean first?

a. Windowsills

b. Bathrooms

c. Floors

d. High surfaces

A

Rationale

d. EVS staff should be cleaning from top to bottom—that is, from high to low surfaces. Cleaning in this manner will help to prevent cross-contamination of already cleaned areas. Floors should, therefore, be cleaned last. Cleaning should also proceed from the cleanest to the dirtiest area, with the bathroom floor being the very last place cleaned.

117
Q

While reviewing a patient’s microbiology results, an infection preventionist (IP) notes that the patient’s sputum is positive for Pneumocystis spp. and the patient has been diagnosed with Pneumocystis pneumonia (PCP). Knowing that PCP is an opportunistic organism, which of the following patient risk factors should the IP be concerned about?

a. Immunocompromising factors

b. Vaccination status

c. Travel history

d. Environmental factors

A

Rationale

a. An opportunistic infection does not usually cause disease in healthy people but may become pathogenic in immunocompromised individuals. The IP should be concerned about immunocompromising factors such as human immunodeficiency virus (HIV) status or other host defects that impair the immune system.

118
Q

During a recent ice storm, the water pipes above the ambulatory surgical center froze and ruptured, causing water intrusion into surgical suites. There is concern that water may also be in some channels behind walls. The water remediation company would like to cut a hole in the wall to assess moisture levels. Which of the following is the infection preventionist (IP) responsible for in this situation?

a. Monitoring for environmental contamination

b. Asking that the remediation company wait to cut into the calls until after all other work is complete

c. Providing respiratory protection to all workers in the location

d. Allowing the remediation company to follow their own protocols

A

Rationale

a. One of an IP’s roles is managing the physical environment to protect patients and workers. Workers should only use the respiratory protection type they are fit tested for.

119
Q

Which of the following quality assurance and performance improvement (QAPI) tools is best suited to analyzing a health system’s pandemic preparedness program?

a. Root cause analysis (RCA)

b. Define, Measure, Analyze, Improve, and Control (DMAIC)

c. Plan-Do-Study-Act (PDSA)

d. Strengths, Weaknesses, Opportunities, and Threats (SWOT) analysis

A

Rationale

d. A SWOT analysis can be used drive change by assessing organizational data though the identification of strengths, weaknesses, opportunities, and threats. DMAIC, PDSA and RCA are also QAPI tools, but DMAIC and PDSA focus on process or data-driven activities, while RCA is used to identify factors that contribute to adverse events. As such, a SWOT analysis is best suited to analyzing a pandemic preparedness program.

120
Q

Which of the following pieces of information should be included in the documentation that the sterile processing department keeps for each sterilization cycle?

a. Include the lot number, date, and time; contents of the load; exposure time; and the name of the person using the processed item.

b. Include the lot number, date, and time; contents of the load, including quantity, department, and a specific description of the items; exposure time and temperature; operator’s name or initials; and the response of the chemical indicators (CI) placed in the process challenge device (PCD).

c. Include aeration information for any cleaning or disinfection product used.

d. Include the operator’s full name and license number, exposure time and temperature, and date and time of instrument use.

A

Rationale

b. For each sterilization cycle, the sterile processing staff should document the following in the sterilizer records:

  • Lot number, date, and time
  • Contents of the load, including quantity, department, and a specific description of the items
  • Exposure time and temperature
  • Operator’s name or initials
  • Results of biological indicator (BI) testing and Bowie–Dick testing, if applicable
  • Response of the CI placed in the PCD
  • Any reports of inconclusive or nonresponsive CIs found later in the load
  • Aeration information (for ethylene oxide [EO]-processed loads)
121
Q

A patient with a severe headache has an abnormal peripheral white blood cell (WBC) count. Which of the following abnormalities would prompt an urgent need for antibiotics?

a. The presence of bands

b. Increased lymphocytes

c. Decreased eosinophils

d. Decreased neutrophils

A

Rationale

a. An increased number of neutrophils in a white cell count typically indicates a bacterial infection. In an acute phase, immature neutrophils, called bands, can be present. Because patient in this example has a severe headache and an indication of a bacterial infection, there would be a concern that the patient may have a meningococcal infection. Meningococcal infection requires immediate antibiotic treatment.

122
Q

Which of the following skin antiseptics is considered to have excellent persistent residual activity?

a. Alcohol

b. Chlorhexidine

c. Iodophor

d. Triclosan

A

Rationale

b. Chlorhexidine is known for its persistent residual activity.

123
Q

Policies and procedures should be based on national standards from recognized infection prevention and control (IPC) agencies or regulatory bodies. Policies and procedures should be reviewed at regular intervals. Which of the following scenarios best describes when an IPC policy and procedures review should occur?

a. When there are changes to IPC committee participants

b. When new policies and procedures are created by the facility leadership

c. Quarterly and as recommended by the participants of the IPC committee

d. Annually and when there is a communicable disease outbreak or change in products or services

A

Rationale

d. Policies and procedures should be reviewed at least annually and revised as needed (for example, when services or products change or a new outbreak occurs). This ongoing review process should be documented because review process documentation is routinely inspected during facility surveys. If a healthcare facility does not perform this review on time, it may be receive a citation from regulatory or accreditation review bodies.

124
Q

Which of the following is synthesized by living organisms to kill microorganisms?

a. Suppressants

b. Antibiotics

c. Supplements

d. Antiseptics

A

Rationale

b. Antibiotics are a kind of antimicrobial synthesized by living organisms to kill microorganisms. Antiseptics are not synthesized by living organisms and are used to treat minor conditions. Suppressants are substances that restrain or suppress something. Supplements are products used to complete a deficiency.

125
Q

An infection preventionist (IP) is collaborating with the nursing and the quality improvement department to evaluate a recent sentinel event. Which of the following types of analysis should they use?

a. Plan-Do-Study-Act (PDSA) analysis

b. Gap analysis

c. Strengths, Weaknesses, Opportunities, and Threats (SWOT) analysis

d. Root cause analysis (RCA)

A

Rationale

d. RCA is a process for identifying the basic or causal factors that underlie variation in performance, including the occurrence or possible occurrence of a sentinel event. An RCA focuses on systems and processes, not on individual performance. An RCA identifies potential improvements that may decrease the likelihood of such events in the future; it may also determines, after analysis, that no such improvement opportunities exist.

126
Q

An infection preventionist (IP) is assessing the risk of illness in the local community, and they are concerned about the large number of homeless individuals near the hospital. They note this on the facility’s risk assessment. What is the rationale for the IP including this information?

a. Proximity between individuals promoting disease transmission

b. Use of recreational drugs among the homeless population

c. Underlying health conditions of homeless individuals

d. Lack of availability of basic hygiene among the homeless population

A

Rationale

d. While recreational drug use, proximity, and underlying conditions all play their roles in disease transmission, they are not exclusively features of homelessness. The lack of availability of basic hygiene is, however, consistent among those experiencing homelessness.

127
Q

A registered nurse (RN) is consistently observed to be out of compliance with a hospital’s hand hygiene (HH) policy. The infection preventionist (IP) notes that there was an increase of multiple-drug-resistant organisms (MDROs) in the same department that this RN works. The RN failing to follow hospital policy can have legal ramifications. This an example of which of the following?

a. Breach of duty

b. Failure to follow standards of practice

c. Failure to follow evidence-based practices

d. Unreasonable protocol

A

Rationale

a. Healthcare providers and facilities have a legal duty to care for their patients. A breach of duty is any deviation from recognized standards of care.

128
Q

An infection preventionist IP is working to evaluate the transportation of reusable, soiled instruments from the outpatient clinic to the sterile processing department. Which of the following methods should the IP use in their evaluation?

a. Plan-Do-Study-Act (PDSA)

b. Gap analysis

c. Pareto analysis

d. Failure mode and effects analysis (FMEA)

A

Rationale

d. FMEA is a systematic, proactive method for evaluating a process to identify where and how it might fail and to assess the relative impact of different failures in order to identify the parts of the process that need change.

129
Q

You notice there are four new C. difficile infections on the same hallway in the telemetry unit. You fear there may be an outbreak. Which of the following steps must be done before you take any other action?

a. Confirm presence of an outbreak.

b. Establish an initial case definition.

c. Alert key partners about the investigation.

d. Perform a literature review.

A

Rationale

a. In simple terms, an outbreak is an increase over the expected occurrence of an event. When a possible outbreak is reported, the initial step in the investigation is to confirm that what is being reported indeed represents an increase in the expected outcome.

130
Q

An infection prevention and control program (IPCP) generally includes:

  • A documented infection prevention and control (IPC) plan.
  • At least one trained infection preventionist (IP).
  • At least one IP per a certain number of hospital beds.
  • An antimicrobial stewardship program.
  • A health promotion program.

Which of the following is another requirement?

a. A licensed food-service program

b. Reduction of healthcare-associated infections (HAIs)

c. A registered environmental services (EVS) program

d. A quality assurance and performance improvement (QAPI) component

A

Rationale

d. There is no requirement that facilities have a licensed food-service or registered EVS program. While it is expected that efforts will be made to prevent HAIs, a reduction in their numbers is not a regulatory requirement. A QAPI component that addresses systems of care, quality of life, and outcomes of care is often a regulatory requirement.

131
Q

To assist with proper patient placement and the correct use of isolation precautions, which of the following pieces of information is it most critical that an infection preventionist (IP) communicate to a post-acute facility upon patient transfer?

a. Patient is colonized with methicillin-susceptible Staphylococcus aureus in their nose.

b. Patient has Candida auris in a leg wound.

c. Patient has a peripherally inserted central catheter (PICC) in place for antibiotic treatment.

d. Patient just completed treatment for an E. coli urinary tract infection (UTI).

A

Rationale

b. While all of the above information may be important to communicate to a post-acute facility, the most important thing to communicate to facilitate proper placement and isolation management is the fact that the patient is positive for Candida auris. C. auris is a multiple-drug-resistant organism (MDRO) that requires contact precautions and is more common in patients who have been hospitalized in a healthcare facility for a long time, have a central venous catheter (CVC) or other lines or tubes entering their body, or have previously received antibiotics or antifungal medications.

132
Q

When comparing glutaraldehyde to hydrogen peroxide and peracetic acid for high-level disinfection, which of the following statements describes a disadvantage that glutaraldehyde has that hydrogen peroxide and peracetic acid do not?

a. It has poor material compatibility.

b. It does not require engineering controls.

c. It is a respiratory irritant.

d. It has a shelf life of six months once activated.

A

Rationale

c. Glutaraldehyde is a respiratory irritant. Healthcare personnel (HCP) may be exposed to elevated levels of glutaraldehyde vapor when equipment is processed in poorly ventilated rooms, when spills occur, when glutaraldehyde solutions are activated or changed, or when open immersion baths are used. Acute or chronic exposure can result in skin irritation or dermatitis, mucous membrane irritation (eye, nose, mouth), or pulmonary symptoms.

133
Q

An infection preventionist (IP) has been asked to assess and evaluate a low-level disinfectant product for a new piece of medical equipment. Which of the following is the most important thing for the IP to consider when making their evaluation?

a. Product delivery

b. Healthcare personnel (HCP) preference

c. Efficacy studies

d. Odor

A

Rationale

c. Ensuring product efficacy for low-level disinfection is the most important consideration. Because the product will be used on medical equipment (and not staff or other persons), product odor and HCP preferences are less important than product efficacy.

134
Q

When comparing data across organizations, which of the following statements describes the most important requirement of the data to be compared?

a. Data must be well-organized and usefully formatted

b. Data must be well-defined

c. Data must be comprehensible and actionable

d. Data must include rates or ratios calculated using the same methodology as a nationally validated surveillance system

A

Rationale

d. Data for benchmarks and comparison with other similar facilities (e.g., data from the National Healthcare Safety Network [NHSN]) must follow specific definitions and algorithms exactly to be valid.

135
Q

After a collaborative planning session with the local health department, a healthcare facility’s infection prevention and control (IPC) department decides to initiate the following programs to promote health and prevent acute disease: 1) vaccination programs, 2) human immunodeficiency virus (HIV) testing, and 3) tuberculosis (TB) and sexually transmitted disease (STD) testing and treatment. What it the first step the infection preventionist (IP) should take in implementing these programs?

a. Perform a thorough study of the patient demographic for the community.

b. Initiate an all-hazards approach.

c. Get approval for funding.

d. Establish infrastructure and legal authority.

A

Rationale

a. Many prevention efforts are targeted to specific populations or subpopulations. As such, the IP must have an understanding of the needs of the population to be served.

136
Q

During a recent hurricane, a hospital experiences significant water intrusion impacting all levels of safe operation. Incident command is established and a decision to evacuate all patients is made. What should the infection preventionist (IP) do first to assist the evacuation?

a. Wait for their role assignment from the incident commander.

b. Identify all patients who require isolation precautions.

c. Ensure that all staff and patients are wearing the highest level of PPE available.

d. Triage patients to prioritize isolation needs and conserve personal protective equipment (PPE).

A

Rationale

b. By first identifying patients who require isolation precautions, the IP can determine the next steps in their evacuation response.

137
Q

A 30-year-old pregnant patient presents to the emergency department with fever, flu-like symptoms, nausea, vomiting, and a complaint of a stiff neck. The emergency department (ED) provider has a working diagnosis of meningitis. The patient has no ill contacts and was feeling well until three days ago. Their history reveals they recently ingested deli meats. Which of the following infectious agents is the most likely cause of the meningitis?

a. Herpes-simplex virus

b. Streptococcus pneumoniae

c. Coxsackievirus

d. Listeria monocytogenes

A

Rationale

d. All of the above infectious agents can cause meningitis; however, Listeria monocytogenes has been found in uncooked meats, uncooked vegetables, unpasteurized milk, foods made from unpasteurized milk, and processed foods. Pregnant women are more likely to contract Listeria spp. Listeriosis is 18 times more common in pregnant women than in the non-pregnant population. 16 to 27 percent of all Listeria infections occur in pregnant women.

138
Q

What type of study design is best suited to identifying the source of a cluster of salmonella infections observed among family members after a weekend reunion?

a. Descriptive

b. Case-control

c. Correlation

d. Cohort

A

Rationale

b. Case-control studies start with the knowledge of who does and does not have the illness. The other types of study designs would not be useful with such a relatively small sample size.

139
Q

A recent study reports relative risks with 95 percent confidence intervals (CI), each comparing a post-intervention infection rate to its pre-intervention baseline: 0.84 [0.70–1.02] for methicillin-resistant Staphylococcus aureus (MRSA) and 0.82 [0.72–0.94] for vancomycin-resistant enterococci (VRE). Which of the following statements about this data is true?

a. MRSA and VRE infection rates were significantly changed by the intervention.

b. MRSA and VRE infection rates were not significantly changed by the intervention.

c. MRSA infection rates were significantly changed by the intervention.

d. VRE infection rates were significantly changed by the intervention.

A

Rationale

d. Only the VRE CI excludes the null hypothesis value (risk ratio = 1). The infection rate reduction was significant only for VRE.

140
Q

The validity of a culture report is dependent on the quality of the specimen sent. To determine whether an expectorated specimen was sputum and not saliva, the Gram stain should show which of the following?

a. Fewer than 10 epithelial cells per low-power field

b. More than 10 epithelial cells per low-power field and moderate to abundant polys

c. More than 10 epithelial cells per low-power field and abundant Pseudomonas aeruginosa in pure culture

d. Many white blood cells (WBCs) and organisms on low-power field

A

Rationale

a. All Gram stains are scanned for the presence or absence of WBCs (indicative of infection) and squamous epithelial cells (indicative of mucosal contamination) that give information about the specimen quality. A sputum specimen showing more than 10 squamous epithelial cells per low-power field, regardless of the number of WBCs, is indicative that the specimen is grossly contaminated with saliva and the culture results cannot be properly interpreted. The presence of fewer than 10 squamous epithelial cells per low-power field indicates that true lower respiratory tract secretions have been collected.

141
Q

An infection preventionist (IP) works in a tribal healthcare facility. Diabetes is a common comorbidity among the population of First Americans served there, and the outpatient diabetes clinic educator has reached out to the IP about providing education and training on new patient glucometers to the care delivery team. Based on this information, which of the following kinds of education should the IP provide?

a. Cultural sensitivity training when caring for First Americans

b. Compliance with hand hygiene (HH) practices

c. Avoidance of reusable fingerstick devices

d. Adherence to transmission-based precautions (TBP)

A

Rationale

c. While HH practice and cultural sensitivity are important, due to the type of clinic (diabetes) and educational needs of staff (glucometer use), the IP should train staff on the importance of safe injection practices—e.g., single-use fingerstick devices and single-patient insulin pens—to promote safe work practices and prevent bloodborne pathogen (BBP) exposures.

142
Q

Which of the following statements defines “minimum inhibitory concentration” (MIC)?

a. It is the lowest concentration of a drug that inhibits infection.

b. It is the lowest concentration of a drug that kills the pathogen.

c. It is the lowest concentration of a drug that kills the host.

d. It is the lowest concentration of a drug that prevents growth of a particular organism.

A

Rationale

d. The MIC is the lowest concentration of a chemical that prevents visible growth of a bacterium.

143
Q

In addition to the facility’s geographic location, the types of care and treatment provided, and regulatory guidelines and requirements, what other information should an infection preventionist (IP) consider when evaluating or creating an annual risk assessment?

a. Population demographics and process data

b. Employee demographics and process data

c. Employee demographics and infection data

d. Population demographics and infection data

A

Rationale

d. A risk assessment should include a comprehensive set of factors that will ensure the best patient outcomes for the population served.

144
Q

Bacterial resistance to antimicrobials develops primarily due to which of the following?

a. Induction by host cells

b. Horizontal gene transfer

c. Vertical transmission

d. Random mutation

A

Rationale

b. The conjugation transfer of R plasmids, also termed “horizontal evolution,” is the most common method by which antibiotic resistance is acquired. In this method, the bacteria need not multiply to spread their plasmid. Instead, the plasmid is moved during conjugation. These plasmids often code for resistance to several antibiotics at once.

145
Q

The probability that a surgical site becomes infected may be determined by the interaction of which of the following clinical variables?

a. Inoculum of bacteria, virulence of the microorganism, and efficiency of host defenses

b. Skill of the surgeon, virulence of the microorganism, and efficiency of host defenses

c. Inoculum of bacteria, virulence of the microorganism, efficiency of host defenses, and adjuvant effects of the microenvironment

d. Skill of the surgeon, virulence of the microorganism, efficiency of host defenses, and adjuvant effects of the microenvironment

A

Rationale

c. By the end of a surgical procedure, every surgical wound will have been contaminated with bacteria. The probability of those wounds becoming infected is determined by four clinical variables: 1) inoculum of bacteria, 2) virulence of the microorganism, 3) efficiency of host defenses, and 4) adjuvant effects of the microenvironment.

146
Q

A significance test based on a small sample may not produce a statistically significant result even if the true value differs substantially from the null value. Which of the following terms describes such a result?

a. Significance level

b. Type 1 error

c. Power

d. Type 2 error

A

Rationale

d. When the null hypothesis is false and the infection preventionist (IP) fails to reject it, they make a type II error. The probability of making a type II error is β, which depends on the power of the test. The IP can decrease their risk of committing a type II error by ensuring that their test has enough power. They can do this by making sure that the sample size is large enough to detect a practical difference when one truly exists.

147
Q

Which of the following is an important prerequisite to creating a surveillance plan that incorporates information relevant to the facility and population served?

a. Isolation precautions

b. Infection prevention and control (IPC) risk assessment

c. Active surveillance

d. Passive surveillance

A

Rationale

b. A facility’s IPC risk assessment helps to define the purpose of surveillance and identify the events to be monitored and the data to be collected—all of which information is need to create a surveillance plan.

148
Q

A hospital is experiencing a scabies outbreak on one of their units. All hospital linen is handled at an outside facility. The infection preventionist (IP) has begun to investigate the outbreak. Which of the following recommendations should the IP make?

a. Provide prophylaxis to all staff in the linen department.

b. Bring all linen services in-house so there is better oversight.

c. No interventions need to be made as commercial washers and dryers provide sufficient heat to kill organisms.

d. Treat all linens with an insecticide.

A

Rationale

c. Commercial washers and dryers provide sufficient heat to kill organisms; as such, it is not necessary to move linen in-house, treat staff with a prophylaxis, or apply insecticides.

149
Q

What type of data is collected to identify relevant precautions?

a. Concurrent

b. Retrospective

c. Surveillance

d. Facilitated

A

Rationale

a. Concurrent data collection is performed in real time while a patient is receiving care within a healthcare facility.

150
Q

Six months after educating staff on how to document catheter care, an infection preventionist (IP) performs an audit of patient charts. What type of evaluation does this audit reflect?

a. Impact

b. Process

c. Content

d. Outcome

A

Rationale

d. Outcome measures indicate the results of a process—in this example, documentation of catheter care. Process measures are used to monitor program implementation (e.g., compliance with hand hygiene [HH]). Impact evaluations assess a program’s effect on participants. Appropriate measures include changes in awareness, knowledge, attitudes, behaviors, or skills. Content analysis is a research tool used to determine the presence of certain words, themes, or concepts within a qualitative data set.